PT28.S1.Q03 - 1000-ticket lottery

beemomo8beemomo8 Alum Member
edited December 2015 in Logical Reasoning 82 karma
one of the answer choices states that it is unreason able to believe that 1000 consecutive coin flips will turn up heads,
so is it logically reasonable to believe that 1000 consecutive coin flips will not turn up heads?
http://7sage.com/lsat_explanations/lsat-28-section-1-question-03/

Comments

  • retaker2014retaker2014 Alum Member
    edited April 2014 99 karma
    Well I hope you realise that the answer is wrong for different reasons but...

    No. This is not a logic statement. But I guess you are trying to form a double negation? Not unreasonable, does not = logically reasonable. Secondly, you can't just negate both terms and preserve the logic of a statement. You have to switch the terms that are sufficient and necessary as well. (A--->(~B), (B)--->(~A).

    The correct answer for the question is NOT C, because 1) the stimulus uses a reasonable premise to draw a reasonable conclusion, while the choice C uses an unreasonable premise to draw a reasonable conclusion. 2) The use of odds in choice C is different, flipping a coin 1000 times vs. the stimulus, 1/1000. Therefore choice C is definitely not 'most similar' in its reasoning to the stimulus.
  • beemomo8beemomo8 Alum Member
    82 karma
    I understand the flaw of the the answer (C). I actually wanted to know the logicality of converting unreasonable to reasonable since J.Y did not immediately eliminated (C) after reading the unreasonable part of the premise and I thought there was a logical connection between being reasonable and unreasonable. Thanks though
  • retaker2014retaker2014 Alum Member
    99 karma
    Ah ok. Sorry if my statements were presumptuous.
Sign In or Register to comment.